LSAT and Law School Admissions Forum

Get expert LSAT preparation and law school admissions advice from PowerScore Test Preparation.

 Administrator
PowerScore Staff
  • PowerScore Staff
  • Posts: 8916
  • Joined: Feb 02, 2011
|
#23346
Complete Question Explanation

Parallel Flaw. The correct answer choice is (D)
The stimulus contains flawed reasoning in the form of a mistaken negation.
  • Citizens Allowed to Communicate
    ..... OR ..... ..... ..... ..... ..... :arrow: ..... Censorship
    Citizens Permitted Access
Therefore,
  • Citizens Allowed to Communicate
    ..... AND ..... ..... ..... ..... ..... :arrow: ..... Censorship
    Citizens Permitted Access
    (if the public itself is unwilling to provide funds)
Since we are seeking an answer which parallels this reasoning, we should look for the choice that contains a mistaken negation.

Answer choice (A): This is not the correct answer since it is a proper contrapositive:
  • Actions that cause Unnecessary Harm ..... :arrow: ..... Just Action
Therefore,
  • Just Action ..... :arrow: ..... Actions that cause Unnecessary Harm
If a just action causes harm, then, it must be necessary since just actions never cause unnecessary harm. This is a valid reasoning, and is not the correct answer.

Answer choice (B): This answer choice does not contain a mistaken negation. The conclusion also does not match that of the stimulus.

Answer choice (C): This choice does not contain a mistaken negation. The conclusion also does not match that of the stimulus.

Answer choice (D): This is the correct answer choice.
It is a mistaken negation which parallels that found in the stimulus.
  • Doer Risks Life to Benefit Another ..... :arrow: ..... Heroic
Therefore,
  • Doer Risks Life to Benefit Another (if the only thing the doer risks is his reputation) ..... :arrow: ..... Heroic
It contains a mistaken negation just as the stimulus does.

Answer choice (E): This answer choice doe not reflect a mistaken negation.
 alee
  • Posts: 57
  • Joined: Mar 21, 2012
|
#3789
Hi!

Firstly, thanks for all the help. I've used both the Logical Reasoning and Games Bibles, and they've been a great help. Currently taking an Advanced LG course which is awesome too. Great since here in Australia we have nothing LSAT prep wise :)

This is about a logical reasoning 'flawed reasoning' question from the Feb 1994 LSAT, Section 1, Q20.

Mentally, I laid out the argument as follows:

Premise: IF citizens are not allowed to communicate what they are ready to communicate at their own expense OR if other citizens are not permitted access other citizens' communications at their own expense THEN censorship exists.

Conclusion: Public unwillingness to provide funds for certain kinds of scientific, scholarly, or artistic activities are not censorship.

Then I tried to pre-phrase the flaw in the reasoning. Roughly I had: 'the argument draws a conclusion about something not being in group X based on information about properties of things that are in group X'. This corresponds to answer D, which is in fact the right answer. I chose D because the premise identifies a necessary property of heroic deeds (risks own life to benefit another), and then draws a conclusion about an action being non heroic.

But is my reasoning/method actually correct?

Cheers!
 Nikki Siclunov
PowerScore Staff
  • PowerScore Staff
  • Posts: 1362
  • Joined: Aug 02, 2011
|
#3793
Yes, your reasoning is correct but it can be simplified as follows:

Premise: NOT allowed to communicate at our own expense --> censorship

Conclusion: allowed to communicate --> NOT censorship

The argument takes the form of a Mistaken Negation. Just because a certain condition is sufficient to define censorship does not mean that this condition is necessarily required for censorship. It is entirely plausible that conditions other than being forbidden from communicating at our own expense can also generate "censorship."

Likewise, in (D) risking your life is sufficient to make you a hero. The conclusion holds that if the only thing you risk is your reputation (i.e. you don't risk your life), that means you are NOT a hero:

Premise: Risk to Own Life --> Hero

Conclusion: No Risk to Own Life --> Not a Hero

This reasoning takes the same form of a Mistaken Negation as the argument contained in the stimulus. It is therefore the correct answer choice.
 Johnclem
  • Posts: 122
  • Joined: Dec 31, 2015
|
#27255
Hi ,
When I did this question ,before looking at the question stem I thought this was a principle question. In that the stimulus was giving us conditions for what is or isn't a censorship . - I was thinking that way because the sufficient conditions used different conditions .. So I guess what I am asking in terms of your diagram above, how do you equate " citizens not allowed to communicate " with "public unwillingness to provide funds "



Originally I had diagramed it this way :

Not allowed to communicate or citizens not allowed access --> censorship exists
Public not providing funds --> censorship does not exist


Thank you
John
 Adam Tyson
PowerScore Staff
  • PowerScore Staff
  • Posts: 5153
  • Joined: Apr 14, 2011
|
#27324
John,

The public unwillingness isn't being equated with not allowing access. Rather, what the author is saying that public unwillingness is NOT the same as disallowing access, and that since the sufficient condition (not allowing access) has NOT happened then it must be true that the necessary condition (censorship) also does not happen. That's a classic mistaken reversal.

The double negative inherent in this one may throw some folks off. The sufficient conditions here are put in the negative, and then we have to negate those negatives, making them positive. A more pure form of the mistaken negation here would be "if citizens are allowed to communicate and to access communication each at their own expense then there is no censorship". The public unwillingness thing is a bit distracting, but it is definitely NOT the same as disallowing communication or access to communication, so it is a type of negation of the sufficient conditions here, and that's good enough for the LSAT.

I hope that helps!
 wilsonc
  • Posts: 5
  • Joined: Sep 22, 2016
|
#30240
Hi Powerscore,

I marked "D" as a loser because it's structure doesn't exactly parallel the stimulus (stimulus has the OR structure in the conditional statement and D does not). Is it not important for structure to match in flaw questions or am I incorrectly interpreting something?

Thank you.

--Christine
 Adam Tyson
PowerScore Staff
  • PowerScore Staff
  • Posts: 5153
  • Joined: Apr 14, 2011
|
#30276
Thanks for the question, Christine, and for bringing up an important distinction between a standard Parallel Reasoning question and a Parallel Flaw question.

In an ordinary Parallel Reasoning question, whether the stimulus is flawed or valid, the goal is to match up everything as closely as possible. You want the same type of reasoning, the same strength of language, the same validity, the same number and type of premises, etc. Order doesn't matter, nor does topic, but everything else should match up as close to perfectly as you can get it. If that were the type of question were were looking at here, your reaction to answer D would be spot on - I too would reject it because it lacked the "or" element of the stimulus (although I would accept it if none of the other answers was better, since we are supposed to pick the best answer even if it is less than perfect).

On Parallel Flaw questions, the approach is different. While it's nice to find most of those same parallels between the stimulus and the correct answer choice, the main focus is on finding an answer that has the same flaw. Structure, strength of language, number of premises - all that gets moved to a back burner. Found a Mistaken Negation in the stimulus? The answer should be the only one that also has a Mistaken Negation. Stimulus had a Part to Whole flaw? Find that (and not the similar Whole to Part flaw) in the correct answer.

In this case, since we are looking for an argument that has the same flaw, rather than one which truly parallels the argument, finding that Mistaken Negation is what matters, and for that reason D is, at the very least, a strong contender. Since none of the other answers have that same flaw, D wins, despite the structural differences. Matching the flaw takes priority.

Great question! Thanks!
 lsat2016
  • Posts: 59
  • Joined: May 29, 2016
|
#36804
Hello,

Is it necessary to match the conclusion?
Stimulus: cannot

AC: is not
 Adam Tyson
PowerScore Staff
  • PowerScore Staff
  • Posts: 5153
  • Joined: Apr 14, 2011
|
#36847
In a normal parallel reasoning question, lsat2016, yes, you should look to match the conclusion to the greatest extent possible. "Is not" and "cannot" are very different conclusions, and in an ordinary parallel reasoning question that would probably be enough for me to throw that answer on the loser pile.

In parallel flaw questions, though, the conclusion might not match, and it's possible that those two would work together. "If Joe could swim he could join the swim team, but he cannot swim so he cannot join the swim team" is a mistaken negation. That would be parallel to "if Nikki is the President of the club then she is on the Executive Board, but she is not the President so she is not on the Executive Board", because that is also a mistaken negation. "Is not" is parallel in this case to "cannot" because they share the same flaw.

So, for standard parallel reasoning questions, look to parallel everything - the conclusion, the premises, the type of reasoning, the validity, and the abstract structure. On parallel flaw questions, focus almost exclusively on finding the same flaw.

Thanks for the question!
 Toby
  • Posts: 33
  • Joined: Jun 05, 2017
|
#37095
Hello!

I am really struggling to wrap my head around the language used in this stimulus. I feel confident that I understand the conditional reasoning in the premise, but I am confused by the language in the conclusion specifically how it relates to the ideas presented in the premise. I do not understand how the phrase "public unwillingness to provide..." is the negation of "if other citizens are not permitted access..."

I would appreciate a breakdown of the language used in this stimulus please! As always, thanks for the help.

Toby

Get the most out of your LSAT Prep Plus subscription.

Analyze and track your performance with our Testing and Analytics Package.